một số chuyên đề về bất đẳng thức ( Luyen thi dai hoc 2016)

204 293 0
một số chuyên đề về bất đẳng thức ( Luyen thi dai hoc 2016)

Đang tải... (xem toàn văn)

Tài liệu hạn chế xem trước, để xem đầy đủ mời bạn chọn Tải xuống

Thông tin tài liệu

ghiên cứu toán học bằng cách đọc sách hoặc từ một nguồn đáng tin cậy:Tìm một cuốn sách toán học tốt hay trang web toán học phù hợp với bạn. Cùng đưa ra các chủ đề hay bài tập khó, tìm kiếm kiến thức liên quan và chia sẻ chúng với mọi người để nhận được những ý kiến đóng góp giúp cho bạn hiểu về nó cặn kẽ hơn. Bạn có thể làm điều này bằng cách học nhóm, đưa ra câu hỏi cho thầy cô ở trên lớp hoặc tham gia vào diễn đàn toán học. Tôi đưa ra cho bạn một lời khuyên: đừng tìm hiểu toán học một mình, hãy chia sẻ nó và bạn sẽ nhận được nhiều điều thú vị.

MỘT SỐ CHUN ĐỀ BẤT ĐẲNG THỨC MATHEDUCARE.COM PHƯƠNG PHÁP DỒN BIẾN Phan Thành Việt Nội dung: Giới thiệu BĐT biến với cực trò đạt đối xứng Dồn biến kó thuật hàm số BĐT biến với cực trò đạt biên BĐT biến Dồn biến hàm lồi Dồn biến giá trò trung bình Đònh lý dồn biến tổng quát Nhìn lại 10 Bài tập Giới thiệu Các bạn thân mến, nhiều số BĐT mà ta gặp có dấu đẳng thức biến số Một ví dụ kinh điển √ Ví dụ 1: (BĐT Cauchy) Cho x, y, z > x + y + z ≥ 3 xyz Có thể nói số lượng BĐT nhiều nhiều bạn thấy điều hiển nhiên Tất nhiên, không hẳn Tuy nhiên, trường hợp đẳng thức không xảy tất biến ta lại thường rơi vào trường hợp khác, tổng quát hơn: có số (thay tất cả) biến Ở dẫn ví dụ chứng minh phần sau Ví dụ 2: (VMO) Cho x, y, z ∈ R, x2 + y + z = Thì 2(x + y + z) − xyz ≤ 10 vò) Trong BĐT dấu "=" xảy x = y = 2, z = −1 (và hoán matheducare.com MATHEDUCARE.COM Có thể nhiều bạn ngạc nhiên biết có bất đẳng thức mà dấu "=" xảy biến khác Ví dụ sau chứng minh phần sau Ví dụ 3: (Jackgarfukel) Cho a, b, c số thực không âm có tối đa số Thì ta có: a 5√ b c √ ≤ a+b+c +√ +√ c+a a+b b+c Ở đây, dấu đẳng thức xảy a = 3b > 0, c = (và dạng hoán vò) Các bạn tự hỏi giá trò chẳng hạn (3, 1, 0) có đặc biệt mà làm cho đẳng thức xảy Một cách trực giác, ta thấy dường điểm đặc biệt có biến Vì giả thiết biến không âm, nên biến gọi biến có giá trò biên Tóm lại, BĐT mà ta gặp, có trường hợp dấu "=" xảy thường gặp: trường hợp tất biến (ta gọi "cực trò đạt tâm"), tổng quát trường hợp có số biến (ta gọi "cực trò đạt có tính đối xứng"), trường hợp khác dấu "=" xảy có biến có giá trò biên (và ta gọi "cực trò đạt biên") Phương pháp dồn biến đặt để giải BĐT có dạng Ý tưởng chung là: ta đưa trường hợp có hai biến nhau, biến có giá trò biên, số biến giảm Do BĐT đơn giản BĐT ban đầu, đặc biệt BĐT biến cách khảo sát hàm biến số ta chứng minh BĐT đơn giản Chính tư tưởng giảm dần số biến nên phương pháp gọi phương pháp dồn biến Bây trình bày kó thuật phương pháp thông qua toán cụ thể Đối tượng quan trọng mà muốn bạn đọc nắm bắt BĐT với biến số Sau đó, mở rộng cho biến trình bày Cuối cùng, đến với phương pháp dồn biến tổng quát cho n biến số, bạn đọc từ kết "cổ điển" tới cải tiến nhỏ sau kết matheducare.com MATHEDUCARE.COM tổng quát Tinh thần xuyên suốt muốn bạn đọc cảm nhận tính tự nhiên vấn đề Qua đó, bạn lý giải "tại sao", để tự bước đường sáng tạo *Ghi chú: Chúng đánh dấu toán theo mục Vì số lượng đònh lý nên không đánh dấu Chúng cố gắng ghi tên tác giả nguồn trích dẫn tất kết quan trọng, ngoại trừ kết BĐT biến với cực trò đạt đối xứng Xin phác họa lại tư tưởng sau Bài toán có dạng f (x, y, z) ≥ với x, y, z biến số thực thỏa mãn tính chất Điều mong muốn có đánh giá f (x, y, z) ≥ f (t, t, z) với t đại lượng thích hợp tùy theo liên hệ x, y, z (ta gọi kó thuật dồn biến nhau) Sau kiểm tra f (t, t, z) ≥ để hoàn tất chứng minh Lưu ý biến chuẩn hóa bước cuối toán với biến Trong mục này, xem xét ví dụ Bài toán (BĐT Cauchy) Cho x, y, z > 0, chứng minh √ x + y + z ≥ 3 xyz Lời giải: Vì BĐT đồng bậc nên cách chuẩn hóa ta giả sử x+y+z = (*) Viết lại toán dạng f (x, y, z) ≥ với f (x, y, z) = − 27xyz Ta điều kiện (*) bảo toàn (tức thấy thay x y t = x+y có t + t + z = 1), nên ta phải xem xét thay đổi xyz Theo BĐT Cauchy với biến (chứng minh đơn giản) xy ≤ t2, nên xyz ≤ t2z Vậy f (x, y, z) ≥ f (t, t, z) Cuối để ý z = − 2t nên ta có: f (t, t, z) = − 27t2 z = − 27t2 (1 − 2t) = (1 + 6t)(1 − 3t)2 ≥ toán chứng minh xong Đẳng thức xảy x = y 3t = 1, nghóa x = y = 1/3, tương đương với x = y = z matheducare.com MATHEDUCARE.COM *Nhận xét: 1) Có thể nhiều bạn bỡ ngỡ với cách chuẩn hóa Chúng xin nói rõ: bí ẩn Nếu thích, bạn hoàn toàn chuẩn hóa theo cách khác, chẳng hạn giả sử xyz = chứng minh f (x, y, z) ≥ với f (x, y, z) = x + y + z − Khi bước dồn biến chứng √ minh f (x, y, z) ≥ f (t, t, z) với t = xy Đề nghò bạn đọc tự lý giải √ lại xét t = xy, sau lời giải ta xét t = x+y hoàn thành chứng minh theo cách 2) Bạn đọc thắc mắc: không cần chuẩn hóa không? Câu trả lời là: được! Thật vậy, hoàn toàn xét toán f (x, y, z) ≥ √ với f (x, y, z) = x + y + z − xyz Khi bước dồn biến chứng minh √ hay t = xy Thực chất, điều f (x, y, z) ≥ f (t, t, z) với t = x+y hoàn toàn dễ hiểu, tương ứng BĐT có điều kiện BĐT không điều kiện (qua kó thuật chuẩn hóa) 3) Chúng nghó bạn đồng ý rằng: toán chuẩn hóa (tức BĐT có điều kiện) "gợi ý" cho cách dồn biến (phải đảm bảo điều kiện), nhiên, ngược lại toán chưa chuẩn hóa (BĐT không điều kiện) có nhiều cách để dồn biến (nói chung, ta chọn cách dồn biến cho bảo toàn "nhiều" biểu thức BĐT - điều tương đương với chuẩn hóa cho biểu thức có dạng đơn giản nhất) Do đó, phối hợp tốt kó thuật chuẩn hóa dồn biến điều cần thiết Tuy nhiên, quen với điều bạn thấy khác biệt đáng kể chúng Bài toán (BĐT Schur) Cho a, b, c ≥ 0, chứng minh rằng: a3 + b3 + c3 + 3abc ≥ a2 (b + c) + b2 (c + a) + c2(a + b) Lời giải: Xét f (a, b, c) = a3 + b3 + c3 + 3abc − a2(b + c) − b2 (c + a) − c2 (a + b) Đặt t = b+c , ta hi vọng: f (a, b, c) ≥ f (a, t, t) Xét d = f (a, b, c) − f (a, t, t) = b + c − a (b − c)2 Ta thấy với a, b, c số không âm tùy ý d ≥ Tuy nhiên, giả sử a = min{a, b, c} ta có d ≥ Khi ta phải matheducare.com MATHEDUCARE.COM chứng minh f (a, t, t) ≥ Nhưng BĐT tương đương với a(a − t)2 ≥ nên hiển nhiên Bài toán chứng minh xong *Nhận xét: Việc giả sử a = min{a, b, c} thủ thuật thường áp dụng để dồn biến Nhắc lại BĐT biến đối xứng ta giả sử a ≤ b ≤ c (hoặc a ≥ b ≥ c), trường hợp BĐT biến hoán vò vòng quanh ta giả sử a = min{a, b, c} (hoặc a = max{a, b, c}) Bài toán Cho a, b, c số thực dương có tích Chứng minh rằng: 1 + + + ≥ a b c a+b+c Hướng dẫn: Nếu toán ban đầu toán quen thuộc, toán khó Với kinh nghiệm thu từ toán 1, nghó tới việc dồn biến theo trung bình nhân để khai thác giả thiết tích ba số Một lời giải theo hướng bạn Yptsoi (Đài Loan) đưa lên diễn đàn Mathlinks, mà sau xin dẫn lại cách vắn tắt √ √ Ta chứng minh f (a, b, c) ≥√f (a,√ bc, bc) giả sử a ≥ b ≥ c Tiếp theo, ta chứng minh f (a, bc, bc) ≥ 5, √ f , x, x ≥ 5, vớ i x = bc x2 BĐT tương đương với (x − 1)2 (2x4 + 4x3 − 4x2 − x + 2) ≥ Vì biểu thức ngoặc thứ hai dương với x > nên chứng minh hoàn tất Đẳng thức xảy a = b = c = Qua ví dụ trên, thấy cách dồn biến trung bình cộng trung bình nhân thật hữu dụng Tuy nhiên, cách dồn biến vô phong phú uyển chuyển Ví dụ sau minh họa cho điều Bài toán 4.(Iran 1996) Chứng minh với a, b, c > thì: (ab + bc + ca) 1 + + ≥ (a + b)2 (b + c)2 (c + a)2 Hướng dẫn: Đây toán khó Các bạn thấy điều qua kiện matheducare.com MATHEDUCARE.COM dấu "=" đạt a = b = c có a = b, c → Các bạn nên thử để thấy cách dồn biến thông thường trung bình cộng trung bình nhân dẫn đến BĐT vô phức tạp Lời giải sau lấy từ ý thầy Trần Nam Dũng, mà nhìn kó bạn thấy mối tương quan, không tính toán mà tư duy, kó thuật chuẩn hóa dồn biến, mà đề cập nhận xét 3) toán Vì BĐT đồng bậc nên ta giả sử ab + bc + ca = (*) Bây ta hi vọng có đánh giá f (a, b, c) ≥ 94 với f (a, b, c) biểu thức thứ hai vế trái BĐT cần chứng minh Ở t phải thỏa liên hệ (*), nghóa t2 + 2tc = Bằng cách giả sử c = min{a, b, c} ta chứng minh f (a, b, c) ≥ f (t, t, c) Cuối cùng, ta kiểm tra f (t, t, c) ≥ 94 Ở bạn đọc thay vào BĐT để thấy: c = 1−t 2t f (t, t, c) = (1 − t2)(1 − 3t2)2 ≥0 4t2(1 + t2) Bài toán chứng minh xong! *Nhận xét: Ở bước cuối, bạn không chuẩn hóa mà quay lại BĐT đồng bậc: + ) ≥ (t + c)2 4t2 2 ⇔ (t + 2tc)(8t + (t + c) ) − 9(t + c)2 t2 ≥ ⇔ 2tc(t − c)2 ≥ (t2 + 2tc)( Cuối đến với ví dụ mà cực trò không đạt tâm, BĐT đối xứng Các bạn thấy rằng, đường phần quan trọng dồn hai biến nhau, sau cực trò đạt tâm hay không điều mấu chốt Bài toán (VMO) Cho x, y, z số thực thỏa mãn: x2 + y + z = Chứng minh rằng: 2(x + y + z) − xyz ≤ 10 Lời giải Đặt f (x, y, z) = 2(x + y + z) − xyz Chúng ta hi vọng có f (x, y, z) ≥ f (x, t, t), t2 = (y + z 2)/2 (*) (chúng nghó bạn đọc tự lý giải điều này) Lưu ý (*) t nhận giá trò, để matheducare.com MATHEDUCARE.COM đònh ý ta xét t ≥ Ta có: d = f (x, y, z) − f (x, t, t) = 2(y + z − 2t) − x(yz − t2) Ta thấy y + z − 2t ≤ yz − t2 ≤ Do để có d ≤ ta cần x ≤ Từ đó, ta giả sử x = min{x, y, z} Xét trường hợp x ≤ Khi ta dồn biến phải chứng minh f (x, t, t) ≤ 10 Thay t = (9 − x2 )/2 ta có: g(x) = f (x, t, t) = 2x + Ta có: g (x) = 2(9 − x2) − x(9 − x2)/2 4x 3x2 − −√ 2 18 − 2x2 Giải ta thấy phương trình g (x) = có nghiệm âm x = −1 Hơn g liên tục g (−2) > > g(0) nên suy g đổi dấu từ dương sang âm qua điểm x = −1 Vậy ∀x ≤ g(x) ≤ g(−1) = 10 ta có điều phải chứng minh Trường hợp đẳng thức đạt x = −1, y = z = Phần lại ta phải giải trường hợp x > 0, tức số x, y, z dương Lúc dấu BĐT thực ta cần đánh giá đơn giản thông qua dồn biến Nếu x ≥ 3/4 f (x, y, z) = 2(x + y + z) − xyz ≤ √ 27 < 10 3(x2 + y + z ) − ( )3 = 27− 64 Nếu x ≤ 3/4 f (x, y, z) = 2(x + y + z) − xyz ≤ 2( √ 2(y + z )+3/4) ≤= 2( 18 +3/4) < 10 Bài toán chứng minh xong! Dồn biến kó thuật hàm số Đây kó thuật quan trọng phương pháp dồn biến Tuy nhiên giới thiệu sau phần nhằm trang bò cho bạn kó thuật cần thiết trước qua mục sau Hơn nữa, nghó quen với bạn phân biệt cực trò đạt tâm hay biên, mục nhẹ nhàng matheducare.com MATHEDUCARE.COM Trong $2 thấy để chứng tỏ f (x, y, z) ≥ f (t, t, z) ta việc xét hiệu d = f (x, y, z) − f (t, t, z) tìm cách đánh giá cho d ≥ Tuy nhiên, dạng BĐT đơn giản, phù hợp với biến đổi đại số Giả sử ta phải làm việc với biểu thức f có dạng, chẳng hạn, như: f (x, y, z) = xk + y k + z k với k > cách biến đổi đại số trở nên cồng kềnh phức tạp Kó thuật hàm số dùng để giải trường hợp Ý tưởng này, chẳng hạn để chứng minh f (x, y, z) ≥ f (x, t, t) với t = (y + z)/2, ta xét hàm: g(s) = f (x, t + s, t − s) với s ≥ Sau chứng minh g tăng với s ≥ (thông thường dùng công cụ đạo hàm tiện lợi), suy g(s) ≥ g(0), ∀s ≥ 0, ta thu điều mong muốn Một ví dụ quen thuộc với bạn dồn biến hàm lồi, nhiên quan sát kó thuật dồn biến bối cảnh tổng quát hơn, vấn đề hàm lồi trở lại mục sau toán với n biến Chúng nhấn mạnh rằng, kó thuật khó, chứa đựng nét tinh tế phương pháp dồn biến Những ví dụ sau thể rõ vẻ đẹp sức mạnh phương pháp dồn biến Bài toán Cho k > a, b, c số không âm có tối đa số Chứng minh rằng: ( a k b k c k ) +( ) +( ) ≥ min{2, k } b+c c+a a+b (∗) Lời giải: Tất nhiên ta cần chứng minh BĐT = 23k ⇔ k = ln3 − (các ln2 bạn suy nghó BĐT cho trường hợp lại dẫn đến BĐT cho trường hợp tổng quát) Chú ý với k đẳng thức xảy hai chỗ a = b = c a = b, c = (và hoán vò) Không tổng quát giả sử a + b + c = b ≥ c ≥ a Đặt t = b+c m = b−c , suy b = t + m, c = t − m, a = − 2t Khi vế trái 2 BĐT cần chứng minh là: f (m) = − 2t 2t k + t+m 1−t−m k + t−m 1+m−t k Vì c ≥ a nên 3t − ≥ m ≥ 0, ≥ b + c = 2t nên 12 ≥ t ≥ 13 Ta khảo sát f (m) miền m ∈ [0, 3t − 1] với t ∈ [ 13 , 12 ] số matheducare.com MATHEDUCARE.COM Ta có: k(t + m)k−1 k(t − m)k−1 − (1 − t − m)k+1 (1 + m − t)k+1 (t + m)k−1 (t − m)k−1 f (m) ≥ ⇔ ≥ (1 − t − m)k+1 (1 + m − t)k+1 k+1 [ln(1 − t − m) − ln(1 + m − t)] ≥ ⇔ g(m) := [ln(t − m) − ln(t + m)] − 1−k f (m) = Tiếp tục khảo sát g, ta có: k+1 1 + + + ≥0 t−m t+m 1−k 1−t−m 1+m−t k+1 2(1 − t) −2t + ≥ (1) ⇔ (t − m)(t + m) − k (1 − t − m)(1 + m − t) g (m) = − Đánh giá k+1 1−k ≥ 2, để chứng minh (1) ta cần chứng minh 2(1 − t) −t + ≥ (1) −m (1 − t)2 − m2 ⇔ u(m) = −t + 4t2 − 3t3 + 3tm2 − 2m2 ≥ ⇔ t2 Thật vậy, u (m) < nên u(m) ≥ u(3t − 1) = 2(3t − 1)(2t − 1)2 ≥ Vậy g(m) đồng biến suy g(m) ≥ g(0) = suy f (m) ≥ suy f (m) ≥ f (0) Nhớ m = b = c = t Cuối cùng, ta cần chứng minh h(t) := f (0) ≥ Viết lại: h(t) = k − 2t 2t +2 t 1−t k Ta khảo sát h(t) miền t ∈ [0, 13 ] Ta có: 2ktk−1 k (1 − 2t)k−1 − ≤0 h (t) = (1 − t)k+1 2k tk+1 ⇔ 2k+1 t2k ≤ [(1 − t)(1 − 2t)]k−1 (2) Trong BĐT cuối, vế trái hàm đồng biến theo t vế phải hàm nghòch biến theo t, lưu ý t ≤ 13 nên để chứng minh (2) ta cần: 2k+1 2k ≤ [(1 − )(1 − )]k−1 3 matheducare.com MATHEDUCARE.COM 3.2 Phương pháp đổi biến p; q; r CÁC VÍ DỤ MINH HỌA V½ dư 15 Cho c¡c sè thüc d÷ìng a; b; c thäa m¢n a + b + c = Chùng minh r¬ng + abc 81(ab + bc + ca) : 27 (Vã Th nh V«n) LÍI GIƒI •p dưng b§t ¯ng thùc Schur, ta câ r p2 ) p(4q = 4q B§t ¯ng thùc c¦n chùng minh t÷ìng ÷ìng vỵi +r 81q 27 Sû dưng b§t ¯ng thùc Schur, ta c¦n chùng minh 4q + 81q 27 4q + 81q 27 , B§t ¯ng thùc tr¶n hiºn nhi¶n óng theo b§t ¯ng thùc AM-GM n¶n ta câ pcm a = b = c = 13 : ¯ng thùc x£y v ch¿ V½ dư 16 Cho c¡c sè khỉng ¥m a; b; c thäa m¢n ab + bc + ca = 1: Chùng minh r¬ng ab + bc + ca + + + a+b b+c c+a 3: (Nguy¹n M¤nh Dơng) LÍI GIƒI Ta câ , , X ab + bc + ca + + + a+b b+c c+a (ab + 1)(c + a)(c + b) 3(a + b)(b + c)(c + a) cyc X (ab + 1)(c2 + 1) 3[(a + b + c)(ab + bc + ca) abc] cyc , (a2 + b2 + c2 ) + ab + bc + ca + abc(a + b + c) + + 3abc , (a + b + c) + abc(a + b + c + 3) + 3(a + b + c) 3(a + b + c) °t p = a + b + c; q = ab + bc + ca = 1; r = abc: B§t ¯ng thùc c¦n chùng minh trð th nh p2 + r(p + 3) , (p N¸u p N¸u 1)(p 3p + 2) + r(p + 3) 0 th¼p b§t ¯ng thùc hiºn nhi¶n óng p 3; ¡p dưng b§t ¯ng thùc Schur, ta câ p3 + 9r 4pq 11 matheducare.com MATHEDUCARE.COM 3.2 Phương pháp đổi biến p; q; r p3 4p ,r CÁC VÍ DỤ MINH HỌA Ta c¦n chùng minh p2 3p + + (p + 3) , p4 + 3p3 , (p B§t ¯ng thùc ci hiºn nhi¶n óng v¼ p p3 + 5p2 p3 4p 13p2 + 15p 2)(p + 5p 18 3p + 9) 2v 3p + = p3 + 4p2 + p 2 + 27 >0 Ta câ pcm ¯ng thùc x£y v ch¿ a = b = 1; c = ho°c c¡c ho¡n V½ dư 17 Cho c¡c sè d÷ìng a; b; c thäa m¢n abc = 1: Chùng minh r¬ng 1 + + +3 a2 b c 2(a + b + c): (Vietnam MO 2006, B) LÍI GIƒI th nh °t x = a; y = 1b ; z = 1c , ta câ xyz = 1, çng thíi êi bi¸n th nh p; q; r, ta câ b§t ¯ng thùc trð p2 2q + , 4q p2 2q M b§t ¯ng thùc tr¶n óng theo b§t ¯ng thùc Schur n¶n ta câ pcm a = b = c = 1: ¯ng thùc x£y v ch¿ V½ dư 18 Cho c¡c sè khỉng ¥m a; b; c; khỉng câ sè n o çng thíi b¬ng 0: Chùng minh r¬ng vỵi måi k 1; ta ln câ p a b c (a + b + c)(ab + bc + ca) k + 1: + + +k b+c c+a a+b a3 + b3 + c3 (Ph¤m Sinh T¥n) LÍI GIƒI êi bi¸n b§t ¯ng thùc theo p; q; r v chu©n hâa cho p = Ta c¦n chùng minh b§t ¯ng thùc 2q + 3r +k q r q 3q + 3r p k+1 Ta câ 2q + 3r +k q r ¯ng thùc x£y (a; b; c) = q 3q + 3r p = 3q + 3r +k q r 1 3q + 3r +k q p p k+2 k 3+ k+1 x; x; q +1 3q + 3r q +1 3q + 3r p k + 1: ho°c c¡c ho¡n t÷ìng ùng Mët sè b i tªp t÷ìng tü 12 matheducare.com MATHEDUCARE.COM 3.2 Phương pháp đổi biến p; q; r B i to¡n Cho c¡c sè khỉng ¥m a; b; c: Chùng minh r¬ng vỵi måi k CÁC VÍ DỤ MINH HỌA 1; ta ln câ a b c (a + b)(b + c)(c + a) + + +k b+c c+a a+b a3 + b3 + c3 p k + 1: (Ph¤m Sinh T¥n) B i to¡n Cho c¡c sè khỉng ¥m a; b; c; khỉng câ sè n o çng thíi b¬ng 0: Chùng minh r¬ng a b c 9(ab + bc + ca) + + + b+c c+a a+b a2 + b2 + c2 6: (Ph¤m Sinh T¥n) V½ dư 19 Cho c¡c sè khỉng ¥m a; b; c; khỉng câ sè n o çng thíi b¬ng 0: Chùng minh r¬ng a b+c b c+a + + c a+b + 10abc (a + b)(b + c)(c + a) 2: (D÷ìng ùc L¥m) LÍI GIƒI °t x = 2a b+c ; y 2b c+a ; z = = 2c a+b , ta câ xy + yz + zx + xyz = B§t ¯ng thùc trð th nh x2 + y + z + 5xyz ÷a b§t ¯ng thùc v· d¤ng p; q; r, tø gi£ thi¸t, ta câ q + r = v b§t ¯ng thùc trð th nh p2 2q + 5r , p2 N¸u 7q + 12 p, sû dưngb§t ¯ng thùc Schur, ta câ )4 p2 ) p(4q r q+ p(4q p2 ) p + 36 4p + ,q ) p2 7q + 12 p2 7(p3 + 36) + 12 4p + N¶n ta ch¿ c¦n chùng minh ÷đc p2 i·u n y óng v¼ N¸u p 4, ta câ p2 p 16 7(p3 + 36) + 12 4p + , (p p 3)(p2 16) 2q p2 3q 3: 4q n¶n p2 Vªy b§t ¯ng thùc ÷đc chùng minh ho¡n t÷ìng ùng 2q + 5r p2 ¯ng thùc x£y x = y = z = ho°c x = y = 2; z = ho°c c¡c 13 matheducare.com MATHEDUCARE.COM 3.2 Phương pháp đổi biến p; q; r CÁC VÍ DỤ MINH HỌA V½ dư 20 Cho c¡c sè khỉng ¥m a; b; c thäa m¢n a + b + c = 3: Chùng minh r¬ng ab + + bc : ca (Vasile Cirtoaje) LÍI GIƒI Chuyºn êi b§t ¯ng thùc v· nh÷ sau 108 3r2 48q + 13pr , 4(9 4q + 3r) + r(1 r) Ta th§y b§t ¯ng thùc tr¶n óng a+b+c r = abc =1 v theo b§t ¯ng thùc Schur th¼ 3r 3p(4q p2 ) = 4q ) 3r + 4q 0: Vªy b§t ¯ng thùc ÷đc chùng minh ¯ng thùc x£y v ch¿ a = b = c = ho°c a = 0; b = c = ho°c c¡c ho¡n t÷ìng ùng V½ dư 21 Cho c¡c sè khỉng ¥m a; b; c; khỉng câ sè n o çng thíi b¬ng 0: Chùng minh r¬ng a2 (b + c) b2 (c + a) c2 (a + b) + + b2 + c2 c + a2 a + b2 a + b + c: (Darij Grinberg) LÍI GIƒI •p dưng b§t ¯ng thùc Cauchy-Schwarz, ta c¦n chùng minh " X a2 (b + c)2 cyc #2 X !" a cyc X # a2 (b + c)(b2 + c2 ) cyc êi bi¸n theo p; q; r, â b§t ¯ng thùc vi¸t th nh r(2p3 + 9r 7pq) •p dưng BDT Schur, ta câ p3 + 9r 4pq v b§t ¯ng thùc quen thc p2 x£y v ch¿ a = b = c ho°c a = b; c = 0: 3q 0, ta câ pcm ¯ng thùc V½ dư 22 Cho c¡c sè khỉng ¥m a; b; c thäa m¢n a + b + c = 1: Chùng minh r¬ng 5(a2 + b2 + c2 ) 6(a3 + b3 + c3 ) + 1: LÍI GIƒI êi bi¸n v· p; q; r; ta c¦n chùng minh 10q 6(1 , 18r 3q + 3r) + 8q + M«c kh¡c, b§t ¯ng thùc tr¶n óng theo b§t ¯ng thùc Schur n¶n ta câ pcm V mët v½ dư iºn h¼nh cho ph÷ìng ph¡p n y l b§t ¯ng thùc Iran 1996 14 matheducare.com MATHEDUCARE.COM 3.2 Phương pháp đổi biến p; q; r CÁC VÍ DỤ MINH HỌA V½ dư 23 Cho c¡c sè khỉng ¥m x; y; z; khỉng câ sè n o çng thíi b¬ng Chùng minh r¬ng 1 + + (x + y)2 (y + z)2 (z + x)2 (xy + yz + zx) : (Iran MO 1996, Ji Chen) LÍI GIƒI Sû dưng ph÷ìng ph¡p êi bi¸n p; q; r, ta chuyºn b§t ¯ng thùc v· d¤ng nh÷ sau q (p2 + q)2 4p(pq (pq r)2 r) 17p2 q + 4q + 34pqr 9r2 Bi¸n êi t÷ìng ÷ìng, rót gån, ta c¦n chùng minh 4p4 q , pq(p3 4pqr + 9r) + q(p4 5p2 q + 4q + 6pr) + r(pq 9r) B§t ¯ng thùc ci óng n¶n ta câ pcm ¯ng thùc x£y v ch¿ x = y = z ho°c x = y; z = ho°c c¡c ho¡n t÷ìng ùng Qua c¡c v½ dư tr¶n, câ l³ c¡c b¤n cơng ¢ ÷đc h¼nh dung ½t nhi·u v· b§t ¯ng thùc Schur v nhúng ùng dưng cõa nâ ph÷ìng ph¡p êi bi¸n p; q; r: º k¸t thóc b i vi¸t n y, míi c¡c b¤n còng gi£i mët sè b i tªp sau B i to¡n Cho c¡c sè khỉng ¥m a; b; c thäa m¢n a3 + b3 + c3 = Chùng minh r¬ng a4 b4 + b4 c4 + c4 a4 3: (Vasile Cirtoaje) B i to¡n Cho c¡c sè khỉng ¥m a; b; c: Chùng minh r¬ng a2 + b2 + c2 + 2abc + 2(ab + bc + ca): (Darij Grinberg) B i to¡n Cho c¡c sè khỉng ¥m a; b; c thäa m¢n a2 + b2 + c2 = Chùng minh r¬ng 12 + 9abc 7(ab + bc + ca): (Vasile Cirtoaje) B i to¡n Cho c¡c sè d÷ìng a; b; c thäa m¢n abc = 1: Chùng minh r¬ng a2 + a + b2 + b + c2 c+1 3: (Vơ ¼nh Q) B i to¡n Cho c¡c sè thüc a; b; c thäa m¢n a2 + b2 + c2 = Chùng minh r¬ng 2(a + b + c) abc 10: (Vietnam MO 2002, Tr¦n Nam Dơng) B i to¡n 10 Cho c¡c sè d÷ìng a; b; c thäa m¢n abc = 1: Chùng minh r¬ng 1+ a+b+c : ab + bc + ca (Vasile Cirtoaje) 15 matheducare.com MATHEDUCARE.COM 3.2 Phương pháp đổi biến p; q; r CÁC VÍ DỤ MINH HỌA B i to¡n 11 Cho c¡c sè d÷ìng a; b; c thäa m¢n abc = 1: Chùng minh r¬ng 2(a2 + b2 + c2 ) + 12 3(a + b + c) + 3(ab + bc + ca) (Balkan MO) B i to¡n 12 Cho c¡c sè khỉng ¥m a; b; c; khỉng câ sè n o çng thíi b¬ng 0: Chùng minh r¬ng vỵi måi k 3; ta p 1 k k+1 p + + + : a+b b+c c+a a+b+c ab + bc + ca (Ph¤m Kim Hòng) B i to¡n 13 Cho c¡c sè khỉng ¥m a; b; c thäa m¢n ab + bc + ca + 6abc = Chùng minh r¬ng a + b + c + 3abc 6: (L¶ Trung Ki¶n, Vã Qc B¡ C©n) B i to¡n 14 Cho c¡c sè khỉng ¥m x; y; z; khỉng câ sè n o çng thíi b¬ng 0: T¼m h¬ng sè a nhä nh§t º b§t ¯ng thùc sau óng x+y+z a xy + yz + zx a (x + y)(y + z)(z + x) : (Ivan Borsenco, Irurie Boreico) B i to¡n 15 Cho c¡c sè d÷ìng a; b; c thäa m¢n abc = 1: Chùng minh r¬ng r 3 a+b+c 10 a + b + c : 3 B i to¡n 16 Cho c¡c sè khỉng ¥m a; b; c thäa m¢n a + b + c = 1: Chùng minh r¬ng 1 + + + 2abc a+b b+c c+a 247 : 54 B i to¡n 17 Cho a; b; c [1; 2]: Chùng minh r¬ng a2 (b + c) + b2 (c + a) + c2 (a + b) 7abc: B i to¡n 18 Cho c¡c sè khỉng ¥m a; b; c thäa m¢n a + b + c = 3: Chùng minh r¬ng ab bc ca + + 1+c 1+a 1+b ab + bc + ca: (Vasile Cirtoaje) CHĨC C•C B„N TH€NH CỈNG!!! 16 matheducare.com MATHEDUCARE.COM Dồn biến cổ điển bất đẳng thức Jack Garfunkel Võ Quốc Bá Cẩn Đại học Y Dược Cần Thơ Ngày tháng năm 2008 Tóm tắt nội dung Trong này, giới thiệu cách chứng minh phép dồn biến cổ điển cho bất đẳng thức sau 5p a+b+c b c a p +p +p c+a a+b b+c Bất đẳng thức tác giả Jack Garfunkel đề nghị tạp chí Crux Magazine năm 1991 (bài tốn 1490) Đây tốn hay khó nhận nhiều lời giải cho lời giải phép dồn biến túy đến chưa nhận Trước hết cần có kết sau làm bổ đề phụ trợ cho chứng minh bất đẳng thức Jack Garfunkel Bài tốn Cho số khơng âm a; b; c; tất khơng đồng thời 0: Chứng minh b c a + + : 4a + 4b + c 4b + 4c + a 4c + 4a + b (Phạm Kim Hùng) Lời giải Chuẩn hóa cho a + b + c = 3; bất đẳng thức trở thành a , a(3 a)(3 b) + b(3 c + b)(3 b a + c c)(3 a) , a b + b c + c a + abc c) + c(3 b 2 (3 a)(3 b)(3 c) Khơng tính tổng qt, giả sử b số hạng nằm a c; ta có c(b a)(b c) matheducare.com MATHEDUCARE.COM ) b2 c + c2 a ) a2 b + b2 c + c2 a + abc abc + bc2 2b (a + c) (a + c) 2b + a + c + a + c = 4: 27 b(a + c)2 = Bất đẳng thức chứng minh xong Đẳng thức xảy a = b = c (a; b; c) (2; 1; 0): Nhận xét Đây bổ đề chặt dùng để giải nhiều tốn khác, bạn ghi nhớ nhé! Ngồi ra, làm mạnh bổ đề sau a2 b + b2 c + c2 a + abc + abc(3 ab bc ca) (Võ Quốc Bá Cẩn) Bây đến giải tốn Bài tốn Cho số khơng âm a; b; c, khơng có số đồng thời 0: Chứng minh 5p a b c p a + b + c: +p +p c+a a+b b+c (Jack Garfunkel) Lời giải Ta xét trường hợp Trường hợp c b a; sử dụng bất đẳng thức Cauchy Schwarz, ta có p a b c +p +p c+a a+b b+c Lại có c b (a + b + c) b c a + + a+b b+c c+a a nên a b c + + a+b b+c c+a = = Nên hiển nhiên p Trường hợp a b a b b c c a + + + 2 a+b b+c c+a (c a)(c b)(b a) 25 < (a + b)(b + c)(c + a) 16 a b c +p +p c+a a+b b+c 5p a+b+c c: matheducare.com MATHEDUCARE.COM Trường hợp 2.1 11 b X a p a+b cyc a; sử dụng bất đẳng thức Cauchy Schwarz, ta có !2 " ! a a+b 4a + 4b + c cyc cyc " # ! X X a(a + b + c) X a = a+ a+b 4a + 4b + c cyc cyc cyc ! ! ! X X a X a = a 3+ a+b 4a + 4b + c cyc cyc cyc X a(4a + 4b + c) # X Theo kết tốn trước, ta có X a 4a + 4b + c cyc Nên ta cần chứng minh X a a+b cyc , (11a2 + 6ab Trường hợp 2.2 a 11 b; 27 16 5b2 )c + (ab + c2 )(11b đặt f (a; b; c) = pa a+b + 5a) pb b+c + (đúng) pc : c+a Vì tốn có đẳng thức xảy a = 3; b = 1; c = nên ý tưởng dồn biến 0, tức chứng minh f (a; b; c) f (a1 ; b1 ; 0) với a1 + b1 = a + b + c: Việc làm nói đơn giản thực hiện, bạn thấy khó biểu thức khó cho ta để đánh giá chúng, "cố chấp" giá trị a1 ; b1 hồi dồn biến khó mà ta phải linh động hơn, tùy theo trường hợp cụ thể mà chọn a1 ; b1 thích hợp ứng với trường hợp Chúng ta xét trường hợp nhỏ sau Trường hợp 2.2.1 a 3b; ta chứng minh p a a+b , a2 (a + b + c) , p a+ c a+b+c a2 + ac + c (a + b) c2 (a + b) (đúng) matheducare.com MATHEDUCARE.COM p Do a b c +p c+a b+c r c b+ 3b nên ta cần chứng minh r c b+ b c p +p b+c 3b + c , b2 c2 2bc + +p b + c 3b + c (b + c)(3b + c) 2bc c2 +p 3b + c (b + c)(3b + c) , Do b c nên 3b + c b + b+c 2(b + c); suy p 2b Lại có b + b+c c 3b + c Từ đây, ta đến f (a; b; c) a b; 3b c(b c) = 2(b + c) 2(b + c)(3b + c) c c f a + ;b + ;0 2 đó, ta chứng minh p a a+b , a2 (a + b + c) , 3b 2(b + c) 2b b+c p (b + c)(3b + c) Trường hợp 2.2.2 3b c bc c + b+c c 2b +p 3b + c (b + c)(3b + c) , b+ p a + 83 c a+b+c a2 + ac + c2 (a + b) 64 c (a + b) + ca(3b 64 b c p +p c+a b+c a) (đúng) r b+ c matheducare.com MATHEDUCARE.COM Tương tự trên, ta cần chứng minh b c p +q b+c c + 52 b b+ c 2bc b2 c2 +q + b+c b + c (b + c)( 52 b + c) , , , Do b r c nên c2 2bc +q +c (b + c)( 52 b + c) 2b 2b c 2b +q +c (b + c)( 52 b + c) b+ c bc c+ b+c b + b+c 28 25 (b + c) = (b + c) (b + c) 16 16 2b 8b )q 5(b + c) (b + c)( b + c) b+c Lại có b + b+c 8b 5(b + c) 2b (b + 10c)(5b 3c) c = 40(b + c)(5b + 2c) +c Vậy nên f (a; b; c) Trường hợp 2.2.3 52 b a 11 b; a + c; b + c; 8 ta chứng minh p a a+b , a2 (a + b + c) , f a + 14 c p a+b+c 25 c (a + b) a2 + ac + 196 25 c (a + b) + ca(5b 196 b c p +p c+a b+c 2a) r b+ (đúng) c 14 matheducare.com MATHEDUCARE.COM Tương tự trên, ta cần chứng minh b c p +q b+c c+ , b2 + b+c , c nên Lại có b + 14 b + c c 14 b+ 11 b 2bc c2 +q +c (b + c)( 11 b + c) 11 b c2 2bc +q +c (b + c)( 11 b + c) 11 b , Do b r c 2b +q +c (b + c)( 11 b + c) 11 b b+ c 14 bc c+ 14 b+c b + b + c 14 25 (b + c) 16 8b )q 5(b + c) 11 (b + c)( b + c) 11 b+c 8b 5(b + c) (b + c) 2b c 33b2 + 160bc 125c2 = 70(b + c)(5b + 2c) +c 11 b Vậy nên f (a; b; c) f a+ c; b + c; 14 14 Như vậy, ta cần xét tốn trường hợp có biến đủ Khơng tính tổng qt, giả sử c = 0: Khi bất đẳng thức trở thành p p a + b a+b 5p a+b p b(a + b) (a + b) p , b(a + b) a + 5b p p , a+b b (đúng): ,a+ Bài tốn giải xong matheducare.com MATHEDUCARE.COM Nhận xét Trong lời giải trên, khơng sử dụng máy tính phụ trợ ta lại chia trường hợp có số lẻ 52 ? Câu trả lời xin dành cho bạn Đây lời giải dài phức tạp gợi mở cho nhiều điều việc sử dụng phép dồn biến Từ xưa đến nay, thường "cổ hữu" số kiểu dồn biến, chẳng hạn f (a; b; c) a+b a+b ; ;c 2 f f (a; b; c) f (a + c; b; 0) c c f (a; b; c) f a + ; b + ; 2 Nhưng điều khơng phải lúc ln có mà có số trường hợp Vì thế, cần linh động phép dồn biến, chẳng hạn f (a; b; c) f (a1 ; b1 ; 0) với a1 + b1 = a + b + c: Đây ý tưởng độc đáo thú vị Phần cuối viết, chúng tơi xin giới thiệu bạn số chứng minh khác mà chúng tơi biết cho tốn đẹp Lời giải Đặt b + c = x2 ; c + a = y ; a + b = z với x; y; z > 0; từ ta a= y2 + z2 x2 0; z + x2 b= y2 0; c= x2 + y 2 z2 Bất đẳng thức cho viết lại thành y2 + z2 z x2 + ,x+y+z+ z + x2 x y2 (x + y + z)(x + x2 + y y y)(y xyz z)(z z2 x) 5p 2(x + y + z ) 5p 2(x + y + z ) Từ đây, khơng tính tổng qt, ta giả sử x z đẳng thức Cauchy Schwarz, ta có p p 2(x2 + y + z ) x + y + z y ; sử dụng bất Nên ta cần chứng minh x+y+z+ By (x + y + z)(x y)(y xyz z)(z G.P Henderson x) p x + y2 + z2 why? matheducare.com MATHEDUCARE.COM , f (x) với , f (t) = 4(z y)t3 +4yz(z t2 yz + 4y + 4y z + 4yz y2 ) 4z 5yz p y2 + z2 t Nếu y = z ta có h xy (x f (x) = i y y; ta có lim f (t) = t! 1; f (0) = 4yz(z y ) > 0; lim f (t) = t!1 Lại có f (z) = f h p yz y + z 4y p p y + z = 2yz 4y y + z i 3z = 5y yz (3y 4z)2 p [...]... F2.F0 ≥ 2 (2 ) Từ (1 ) và (2 ) suy ra F4 < F3 < F2 < F1 < F0 = 16 (3 ) Từ (3 ) ta F1 có F4 < 16 suy ra max(a, b, c, d) < 2 Để dẫn tới mâu thuẫn với (3 ), ta sẽ chứng minh F3 ≥ F1 (4 ) Thật vậy: (4 ) ⇔ (1 − a + a2 )(1 − b + b2 )(1 − c + c2 )(1 − d + d2 ) ≥ 1 3 (2 a − 1)2 3 (2 b − 1)2 3 (2 c − 1)2 3 (2 d − 1)2 )( + )( + )( + )≥1 ( + 4 4 4 4 4 4 4 4 (2 a − 1)2 (2 b − 1)2 (2 c − 1)2 (2 d − 1)2 4 ⇔ (1 + )(1 + )(1 + )(1 + )≥... minh rằng: (i) 8 1(1 + a2 )(1 + b2 )(1 + c2 ) ≤ 8(a + b + c)4 (ii) 6 4(1 + a3 )(1 + b3 )(1 + c3 ) ≤ (a + b + c)6 Lời giải: (i) Đặt f (a, b, c) = 8(a + b + c)4 − 8 1(1 + a2 )(1 + b2 )(1 + c2 ) Ta có thể giả sử a ≥ b Xét hàm số g(t) = f (ta, b/t, c) với t ∈ [ b/a, 1] Ta có: g (t) = 32(a − b b b b 3 + c) )(1 + c2 ) )(ta + − 81(a − )(ta + t2 t t2 t Vì t ∈ [ b/a, 1] nên g (t) ≥ 0 nếu: 32(d + c)3 ≥ 81d(1 + c2 )... BĐT(6) ta có: (1 + x )(1 + t ) ≥ 1 + x+t 2 (1 + y 2 )(1 + z 2) ≥ 1 + y+z 2 2 2 2 2 (7 ) 2 2 (8 ) nhân (7 ) và (8 ) vế theo vế suy ra: 2 2 2 2 (1 + x )(1 + t )(1 + y )(1 + z ) ≥ x+t 2 ) 1 +( 2 Từ x + t < 2 và y + z < 2 suy ra: BĐT(6) ta được: x+t 2 ) 1 +( 2 x+t 2 y+z 2 ) ≥ 1+ 1 +( 2 + 2 y+z 2 ) 1 +( 2 y+z 2 (9 ) < 2 Do đó lại áp dụng x+y+z+t 4 2 2 (1 0) Từ (9 ) và (1 0) suy ra: 2 2 2 2 (1 + x )(1 + y )(1 + z )(1 ... cần chứng minh là: c(s + t) s2 − t2 c(s − t) + + f (t) := 3 + (s + t)2 3 + (s − t)2 3 + c2 Ta khảo sát f (t) trên miền t ∈ [0, s − c] Ta có: −c 2c(s2 − t2) c 2c(s2 − t2 ) 2t − + + − 2 2 2 2 2 2 3 + (s + t) (3 + (s + t) ) 3 + (s − t) (3 + (s − t) ) 3 + c2 4cst 8cst(s2 − t2 )(u + v) 2t + = − < 0, ∀t ∈ (0 , s − c) ( ) uv u2 v 2 3 + c2 f (t) = với u = 3 + (s + t)2, v = 3 + (s − t)2 (BĐT (* ) sẽ chứng minh... Ta có 2 2 ( ) ⇔ √ 1 − 2t t−s 5 t+s +√ + √ ≤ 4 s+1−t 1−t−s 2t (1 ) Đặt f (s) = V T (1 ) với s ∈ [0, t], ta sẽ chứng minh f (s) ≤ max(f (0 ), f(t)) Ta có : f (s) = √ t+s 1 1 − 2t 1 − + −√ 3/2 3/2 2(1 − t − s) s + 1 − t 2(s + 1 − t) 2t Vì chưa xác đònh được dấu của f (s) nên ta đạo hàm tiếp f (s) = − f (s) = = 1 3(t + s) 3(1 − 2t) + + 3/2 5/2 (s + 1 − t) 4(s + 1 − t) 4(1 − t − s)5/2 9 15(t + s) 1 5(1 − 2t)... − c] thì: f (t) ≤ f (0 ) = s2 2s(3 − 2s) s2 2cs + = + =: g(s) 3 + s2 3 + c2 3 + s2 3 + (3 − 2s)2 (1 ) Xét g(s) với s ∈ [1, 32 ] Ta có: g (s) = 24s − 12s2 18 − 24s − 6s2 108(s2 − 3s + 4)(s − 1)2 ( s2 − 3s + 6) + = (3 + (3 − 2s)2 )2 (3 + s2 )2 [3 + (3 − 2s)2 ]2 [3 + s2 ]2 √ √ Dễ thấy s2 − 3s + 4 > 0 và −s2 − 3s + 6 = ( 33−3 − s)(s + 33+3 ) nên g (s) 2√ 2 33−3 3 dương trên (1 , s0) và âm trên (s0 , 2 ) với... 32(d + c)3 > 32d(d2 +2dc+3c2 ) ≥ 32d(3 d4 c2 +3c2 ) > 81d(1 +c2 ) (lưu ý là d2 c ≥ 4) với t ∈ [ b/a, 1] Do đó: g(1) ≥ g( b/a) Vậy f (a, b, c) ≥ Vậy g (t) ≥ 0 √ √ f (s, s, c) với s = ab Thay s = 1/ c ta được: 2 1 1 1 f (s, s, c) = f ( √ , √ , c) = 8( √ + c)4 − 8 1(1 + ) 2(1 + c2) c c c c √ c−1 2 5 9 9 5 =( ) (8 c + 16c 2 + 24c4 + 96c 2 + 87c3 + 78c 2 + c √ 3 +99c2 + 120c 2 − 21c + 94 c + 47) ≥0 ( pcm) Đẳng. .. Chứng minh rằng: (a3 + b3 + c3 )(a3b3 + b3c3 + c3 a3) ≤ 36(ab + bc + ca) Lời giải: Không mất tổng quát có thể giả sử a ≥ b ≥ c Đặt f (a, b, c) = 36(ab + bc + ca) − (a3 + b3 + c3)(a3 b3 + b3 + c3 + c3 a3) Khi đó f (a, b + c, 0) = 36a(b + c) − (a3 + (b + c)3)a3(b + c)3 Ta sẽ chứng minh rằng f (a, b, c) ≥ f (a, b + c, 0) Thật vậy, chú ý rằng: 36(ab + bc + ca) ≥ 36a(b + c) và (a3 + b3 + c3 )(a3b3 + b3 c3... vế trái là một hàm theo xi, ta sẽ chứng tỏ: f (xi ) ≤ max{f (p), f(q)} , và dấu bằng xảy ra khi và chỉ khi xi ∈ {p, q} Ta có: f (x) = Ax + Bx + C Có thể khảo sát hàm để ra ngay kết quả (suy ra luôn dấu bằng xảy ra khi xi ∈ {p, q}) Song ở đây trình bày một cách sơ cấp hơn Để ý: B f (xi ) − f (p) = (xi − p)(A − ) xi p B f (xi ) − f (q) = (xi − q)(A − ) xi q Từ đó nếu f (xi ) > max{f (p), f(q)} thì rõ... trên Diễn Đàn Mathlinks Nhắc lại là f (a, b, c, d) = ab(c + d − kcd) + cd(a + b) Đặt g(x) = ab(c + d − kcd) + cd(a + b) thì g là hàm tuyến tính, và ab ∈ [0, t2] (với t = a+b ) nên 2 2 g(ab) ≤ max{g(0), g(t )} Chú ý g(0) = f (0 , a + b, c, d) Vậy ta có: f (a, b, c, d) ≤ max{f (0 , a + b, c, d), f(t, t, c, d)} (2 ) 24 matheducare.com MATHEDUCARE.COM Với cách viết trong BĐT (2 ) ở trên thì việc cực trò đạt tại

Ngày đăng: 28/05/2016, 13:49

Từ khóa liên quan

Tài liệu cùng người dùng

Tài liệu liên quan